The value of a car after it is purchased depreciates according to the formula V(n)=28000(0.875)" where V(n) is the car's value in the nth year since it was purchased. How much value does it lose in its fifth year? [3]

Answers

Answer 1

The given formula for the car's value after n years since it was purchased is V(n) = 28000(0.875)^n. We are asked to find the amount of value the car loses in its fifth year.

To calculate the value lost in the fifth year, we need to find the difference between the value at the start of the fifth year (V(5)) and the value at the end of the fifth year (V(4)).

Using the formula, we can calculate V(5):

V(5) = 28000(0.875)^5

To find V(4), we substitute n = 4 into the formula:

V(4) = 28000(0.875)^4

To determine the value lost in the fifth year, we subtract V(4) from V(5):

Value lost in fifth year = V(5) - V(4)

Now, let's calculate the values:

V(5) = 28000(0.875)^5

V(5) ≈ 28000(0.610)

V(4) = 28000(0.875)^4

V(4) ≈ 28000(0.676)

Value lost in fifth year = V(5) - V(4)

≈ (28000)(0.610) - (28000)(0.676)

≈ 17080 - 18928

≈ -1850

The negative value indicates a loss in value. Therefore, the car loses approximately $1,850 in its fifth year.

To learn more about car - brainly.com/question/20842520

#SPJ11


Related Questions

"Please provide a complete solution.
Use chain rule to find ƒss ƒor ƒ(x,y) = 2x + 4xy - y² with x = s + 2t and y=t√s."

Answers

Answer: To find the total derivative ƒss of ƒ(x, y) = 2x + 4xy - y² with respect to s, where x = s + 2t and y = t√s, we can use the chain rule. The chain rule states that if z = ƒ(x, y) and both x and y are functions of another variable, say t, then the total derivative of z with respect to t can be calculated as:

dz/dt = (∂ƒ/∂x) * (dx/dt) + (∂ƒ/∂y) * (dy/dt)

Let's find ƒss step by step:

Calculate ∂ƒ/∂x:

Taking the partial derivative of ƒ with respect to x, keeping y constant:

∂ƒ/∂x = 2 + 4y

Calculate dx/dt:

Given that x = s + 2t, we can find dx/dt by taking the derivative of x with respect to t, treating s as a constant:

dx/dt = d(s + 2t)/dt = 2

Calculate ∂ƒ/∂y:

Taking the partial derivative of ƒ with respect to y, keeping x constant:

∂ƒ/∂y = 4x - 2y

Calculate dy/dt:

Given that y = t√s, we can find dy/dt by taking the derivative of y with respect to t, treating s as a constant:

dy/dt = d(t√s)/dt = √s

Now, we can substitute these values into the chain rule equation:

dz/dt = (∂ƒ/∂x) * (dx/dt) + (∂ƒ/∂y) * (dy/dt)

= (2 + 4y) * (2) + (4x - 2y) * (√s)

Substituting x = s + 2t and y = t√s, we get:

dz/dt = (2 + 4(t√s)) * (2) + (4(s + 2t) - 2(t√s)) * (√s)

= 4 + 8t√s + 4s√s + 4s + 8t√s - 2t√s√s

= 4 + 12t√s + 4s√s + 4s - 2ts

Therefore, the total derivative ƒss of ƒ(x, y) = 2x + 4xy - y² with respect to s is:

ƒss = dz/dt = 4 + 12t√s + 4s√s + 4s - 2ts

The second partial derivative (ƒss) of ƒ(x, y) = 2x + 4xy - y² with respect to x and y can be found using the chain rule.


To find ƒss, we first need to compute the first partial derivatives of ƒ(x, y) with respect to x and y.

∂ƒ/∂x = 2 + 4y
∂ƒ/∂y = 4x - 2y

Next, we substitute x = s + 2t and y = t√s into the partial derivatives.

∂ƒ/∂x = 2 + 4(t√s)
∂ƒ/∂y = 4(s + 2t) - 2(t√s)

Finally, we differentiate the expressions obtained above with respect to s.

∂²ƒ/∂s² = 4t/√s
∂²ƒ/∂s∂t = 4√s
∂²ƒ/∂t² = 4

Therefore, the second partial derivative ƒss = ∂²ƒ/∂s² = 4t/√s.


Learn more about Chain rule click here :brainly.com/question/29498741

#SPJ11

If f(x)= 10x2 + 4x + 8, which of the following represents f(x + h) fully expanded and simplified? a. 10x2 + 4x+8+h b.10x2+2xh+h2 + 4x + 4h + 8 c. 10x2 + 20xh + 10h2 + 4x + 4h + 8 d.10x2+ 10h² + 4x + 4h + 8
e. 10x2 + 2xh + h2 +4x + h + 8

Answers

The given function is [tex]`f(x) = 10x^2 + 4x + 8`[/tex]. We need to find `f(x + h)`.The formula for [tex]`f(x + h)` is: `f(x + h) = 10(x + h)^2 + 4(x + h) + 8`[/tex].

This can be simplified as follows:[tex]f(x + h) = 10(x^2 + 2xh + h^2) + 4x + 4h + 8f(x + h) = 10x^2 + 20xh + 10h^2 + 4x + 4h + 8[/tex]Therefore, the option (c) is the correct one as it represents `f(x + h)` fully expanded and simplified.

The expanded and simplified form of [tex]`f(x + h)` is `10x^2 + 20xh + 10h^2 + 4x + 4h + 8`[/tex].Hence, the answer to this question is option (c).

In the given problem, we were given a quadratic function. The expression `f(x + h)` is an example of a shifted function. It means that we're changing `x` to `x + h`.

The process is known as horizontal translation or horizontal shift. It's a transformation of the function along the x-axis.

To know more about quadratic function visit -brainly.com/question/29775037

#SPJ11

1. Markov chains (a) Assume a box with a volume of 1 cubic metre containing 1 red particle (R) and 1 blue particle (B). These particles are freely moving in the box and we assume that they are perfectly mixed. We know that when they collide, blue and red particle stick to one another and form a compound particle RB. After a certain amount of time, RB decays again into one R and one B particle. R do not stick to R particles and B particles do not stick to B. After observing the system for a long time, we note that the RB particles remain together on average for 4 seconds before they decay. Equally, on average we wait for 1 second before particles R and B bind. Assume now that we have a box with 2 cubic metres volume and we seed the system with 3 R and 3 B particles. Interpret this system as a Markov chain assuming that particles of the same type are indistinguishable. Draw the transition diagram. In your answer, make sure that you make clear what each state means, and that you label the edges with the transition rates.

Answers

A Markov chain is a stochastic process in which the likelihood of an event happening is dependent solely on the outcome of the previous event. In a Markov chain, the future is independent of the past given the present.

Here, the Markov chain is described as a system that includes 1 red particle (R) and 1 blue particle (B) in a 1 cubic meter box.

When the R and B particles collide, they stick together and form a compound particle RB, which decays after a period of time into one R and one B particle.

The R particles do not adhere to other R particles, and the same is valid for B particles, which do not adhere to other B particles.

We observe that, on average, the RB particles stay together for 4 seconds before decaying, and the R and B particles stick together after waiting for 1 second.

We then consider a 2 cubic meter box containing 3 R and 3 B particles. This system can be interpreted as a Markov chain, with the states being the number of R and B particles.

The state is labeled by the number of red and blue particles present in the system at any given time, such as (2, 3) refers to the state with two red and three blue particles present in the box.

If we start with (3, 3), we can move to either (2, 3) or (3, 2) with equal probability.

The corresponding transition rate would be $3/2$ seconds per transition. After that, we could move to either (2, 2) or (1, 3) or (3, 1), with the corresponding transition rate being $3/4$ seconds per transition.

Finally, we could move to (2, 3) or (3, 2), with the corresponding transition rate being 4 seconds per transition. This is how the system can be interpreted as a Markov chain.

To know more about Markov chain, refer

https://brainly.com/question/25816915

#SPJ11








OF 4. Express the confidence interval 14.26± 3.2 as an interval. 1 POINTS

Answers

The confidence interval 14.26 ± 3.2 can be expressed as an interval by subtracting and adding the margin of error to the point estimate. In this case, the point estimate is 14.26.

The margin of error is 3.2. To calculate the interval, we subtract and add the margin of error from the point estimate:

Lower Bound = 14.26 - 3.2 = 11.06

Upper Bound = 14.26 + 3.2 = 17.46

Therefore, the confidence interval is [11.06, 17.46]. This means that we are 95% confident that the true value lies within this interval.

A confidence interval is a range of values within which we estimate the true population parameter to lie based on a sample. In this case, we have a point estimate of 14.26 and a margin of error of 3.2. The point estimate, 14.26, represents the sample mean or the best estimate we have for the population parameter we are interested in. It is the center of the confidence interval.

The margin of error, 3.2, is the amount of variability or uncertainty associated with the point estimate. It indicates how much the estimate might vary if we were to take multiple samples. A larger margin of error implies a wider interval and more uncertainty. To express the confidence interval, we add and subtract the margin of error from the point estimate. The lower bound, calculated by subtracting the margin of error from the point estimate, represents the minimum value in the interval. The upper bound, obtained by adding the margin of error to the point estimate, represents the maximum value in the interval.

The resulting interval, [11.06, 17.46], indicates that we are 95% confident that the true population parameter lies within this range.

To learn more about confidence - brainly.com/question/25549885

#SPJ11

The observed numbers of days on which accidents occurred in a factory on three successive shifts over a total of 300 days are as shown below. Your boss wants to know if there is a systematic difference in safety that is explained by the different shifts. (20 pts) an Days with Days without an Total Shift Accident Accident Morning 4 96 100 Swing Shift 8 92 100 Night Shift 90 100 Total 22 278 300 a. What are the null and alternative hypotheses you are testing? 10 b. Determine the appropriate test statistic for these hypotheses, and state its assumptions. c. Perform the appropriate test and determine the appropriate conclusion.

Answers

The question examines the difference in safety among three shifts in a factory based on the observed accident counts. It asks for the null and alternative hypotheses, the appropriate test statistic, and the conclusion.

a. The null hypothesis (H₀) would state that there is no systematic difference in safety among the shifts, meaning the accident rates are equal. The alternative hypothesis (H₁) would suggest that there is a significant difference in safety among the shifts, indicating unequal accident rates.

b. To test the hypotheses, a chi-square test for independence would be appropriate. The test statistic is the chi-square statistic (χ²), which measures the deviation between the observed and expected frequencies under the assumption of independence. The assumptions for this test include having independent observations, random sampling, and an expected frequency of at least 5 in each cell.

c. By performing the chi-square test on the observed data, comparing it to the expected frequencies, and calculating the chi-square statistic, we can determine if there is a significant difference in safety among the shifts. Based on the calculated chi-square statistic and its corresponding p-value, we can make a conclusion. If the p-value is below the chosen significance level (e.g., α = 0.05), we reject the null hypothesis and conclude that there is a significant difference in safety among the shifts. If the p-value is above the significance level, we fail to reject the null hypothesis, indicating insufficient evidence to conclude a significant difference in safety among the shifts.

Learn more about hypothesis here:

https://brainly.com/question/29576929

#SPJ11

Find the equation of the line through (4,−8) that is
perpendicular to the line y=−x7−4.
Enter your answer using slope-intercept form.

Answers

The equation of line through (4,−8) that is perpendicular to the line y=−x/7−4 is y = 7x - 36, which is in slope-intercept form.

We need to find the equation of the line through (4,−8) that is perpendicular to the line

y=−x/7−4.

The given line equation is

y = −x/7 − 4.

To find the slope of this line, we need to transform the given equation to slope-intercept form:

y = mx + b where m is the slope and b is the y-intercept.

So, y = -x/7 - 4 can be written as

y = -(1/7)x - 4

Comparing with y = mx + b, we get

m = -1/7

To find the slope of a line perpendicular to this line, we use the relationship that the product of the slopes of two perpendicular lines is equal to -1.

So, the slope of the perpendicular line will be the negative reciprocal of -1/7.

Slope of perpendicular line

= -1/(m)

= -1/(-1/7)

= 7

So, the slope of the required line is 7 and it passes through the point (4, -8).

Using point-slope form, the equation of the line is given by:

y - y1 = m(x - x1)

Substituting m = 7, x1 = 4, and y1 = -8, we get:

y + 8 = 7(x - 4)

Simplifying the equation,

y + 8 = 7x - 28

y = 7x - 36

Know more about the equation of line

https://brainly.com/question/13763238

#SPJ11

(b) The time-dependence of the logarithm y of the number of radioactive nuclei in a sample is given by
y = yo - Xt,
where A is known as the decay constant. In the table y is given for a number of values of t. Use a linear fit to calculate the decay constant of the given isotope correct to one decimal. (8)
t (min) 1 2 3 4
y 7.40 7.35 7.19 6.93

Answers



To calculate the decay constant, you need to perform the linear regression analysis and find the slope of the best-fit line using the given data.

To calculate the decay constant of the given isotope using a linear fit, we can use the equation y = yo - Xt, where y represents the logarithm of the number of radioactive nuclei and t represents time. We have the following data:

t (min): 1   2   3   4
y:         7.40 7.35 7.19 6.93

We can rewrite the equation as y = mx + c, where m is the slope and c is the y-intercept. Rearranging the equation, we get X = (yo - y) / t.

Using the given data, we can calculate the values of X for each time interval:

X1 = (yo - y1) / t1 = (yo - 7.40) / 1
X2 = (yo - y2) / t2 = (yo - 7.35) / 2
X3 = (yo - y3) / t3 = (yo - 7.19) / 3
X4 = (yo - y4) / t4 = (yo - 6.93) / 4

We want to find the value of A, the decay constant, which is equal to -m (the negative slope). To find the best-fit line, we need to minimize the sum of squared errors between the observed values of X and the values predicted by the linear fit.

By performing a linear regression analysis using the data points (t, X), we can obtain the slope of the best-fit line, which will be -A. Calculating the slope using linear regression will give us the value of A.

To calculate the decay constant, you need to perform the linear regression analysis and find the slope of the best-fit line using the given data.

 To  learn more about decimal click here:brainly.com/question/29765582

#SPJ11

A two-tailed test at a 0.0873 level of significance has z values of ____
a. -0.86 and 0.86
b. -0.94 and 0.94
c.-1.36 and 1.36
d. -1.71 and 1.71

Answers

A two-tailed test at a 0.0873 level of significance has z-values of -1.71 and 1.71 (Option D).

What is a two-tailed test?

A two-tailed test is a statistical hypothesis test in which the critical area of a distribution is two-sided and checks whether a sample is significantly different from both ends of the range. This test is used in situations where the difference or deviation from the null hypothesis is unknown or undefined. It is often used when comparing the means of two samples.

The significance level is also known as alpha (α). It determines the probability of a type 1 error. The value of alpha is set before the test begins. It is typically set at 0.1, 0.05, or 0.01. The test's null hypothesis is rejected if the calculated probability is less than or equal to the alpha level.

The correct answer is Option D.

Learn more about z-values here: https://brainly.com/question/30076881

#SPJ11




If Find the value of x+y.. Attachments (n-1)! Σ 69.70.71.....(68+n) X y

Answers

Given a series with the formula (n-1)! Σ 69.70.71.....(68+n) X y.

We need to find the value of x+y.

We are given that the sum of a series can be represented in the form of the first term multiplied by the common ratio raised to the power of the number of terms divided by the common ratio minus 1.

Mathematically, it can be represented as:

[tex]S = a(rⁿ - 1) / (r - 1)[/tex]

Where, S = Sum of seriesa = First termm = Number of termsn = m - 68r = Common ratio For the given series, we can observe that the first term is 69, and the common ratio is 1 as the difference between each consecutive term is 1.

Hence, the sum of the series can be represented as:S = a(m) = 69(m - 68)

Also, we are given that the sum of the series is equal to (n-1)! X y.

Substituting the value of S in the above equation,

we get:(n-1)! X y = 69(m - 68)

Solving the above equation,

we get:

m = (y + 68)

Putting this value of m in the equation of S,

we get:S = 69(y + n)

Therefore, the value of x + y is equal to 69.

Hence, the answer is 69 only in 100 words.

To know more about sum of a series visit:

https://brainly.com/question/8879163

#SPJ11

Consider the following matrices: 2 2 4 A = 2 B = 4 C = 10 -3 -8 For each of the following matrices, determine whether it can be written as a linear combination of these matrices. If so, give the linear combination using the matrix names above. < Select an answer > V₁ = < Select an answer > V₂ = < Select an answer > V3= -16 -32 24 2 10

Answers

Therefore, the linear combination of `A`, `B`, and `C` that can be used to write `V3` is:8/529 A + 24/529 B - 128/529 C.

Given matrices are `A`, `B`, and `C`, and a matrix `V3`.

The question asks if matrix `V3` can be written as a linear combination of `A`, `B`, and `C`.

To do this, we need to solve a system of linear equations. Let's write the system of linear equations to solve for the coefficients of `A`, `B`, and `C` that can be used to write `V3` as a linear combination of the three matrices.

Let `k1`, `k2`, and `k3` be the coefficients of `A`, `B`, and `C`, respectively.

Then, we have: k1A + k2B + k3C = V3

So, the matrix equation becomes: 2k1 + 4k2 + 10k3 = -1610

k1 - 3k2 - 8k3 = 32

To solve this system of linear equations, we can use the matrix method.

First, we write down the coefficient matrix of the system, which is: 2 4 1010 -3 -8

Then, we write down the augmented matrix of the system, which is formed by appending the constant terms of the system to the right of the coefficient matrix: 2 4 10 -1610 -3 -8 32

Next, we perform elementary row operations on the augmented matrix until it is in row echelon form. Using elementary row operations, we can add -5 times row 1 to row 2:2 4 10 -1610 -23 -18 72

We can then multiply row 2 by -1/23 to get a 1 in the second row, second column:2 4 10 -1610 1 3/23 -72/23

Next, we can add -10 times row 2 to row 1:2 0 2/23 16/23-1 1 3/23 -72/23

Finally, we can multiply row 1 by 23/2 to get a 1 in the first row, first column:1 0 1/23 8/23-1 1 3/23 -72/23

So, the solution to the system of linear equations is:

k1 = 1/23(8/23)

= 8/529k2

= 3/23(8/23)

= 24/529k3

= -16/23(8/23)

= -128/529

Thus, we can write matrix `V3` as a linear combination of matrices `A`, `B`, and `C`.

We have given a matrix V3 and three matrices, A, B, and C. We need to find whether matrix V3 can be written as a linear combination of matrices A, B, and C or not.

In order to find whether matrix V3 can be written as a linear combination of matrices A, B, and C or not, we need to solve the following system of linear equations:k1A + k2B + k3C = V3Here, k1, k2, and k3 are the coefficients of matrices A, B, and C, respectively.

Now, we have to solve this system of linear equations in order to find the values of k1, k2, and k3. Once we have found the values of k1, k2, and k3, we can write matrix V3 as a linear combination of matrices A, B, and C. To solve the system of linear equations, we use the matrix method. We first write down the coefficient matrix of the system, which is formed by taking the coefficients of k1, k2, and k3. We then write down the augmented matrix of the system, which is formed by appending the constant terms of the system to the right of the coefficient matrix. We then perform elementary row operations on the augmented matrix to get it into row echelon form. Once the augmented matrix is in row echelon form, we can easily read off the values of k1, k2, and k3 from the matrix.

To know more about matrices visit:

https://brainly.com/question/30646566

#SPJ11

A. quadratic function r is given f(x) = x^2+6x-1
(a) Express f in standart form
f(x) =
(b) find the vertex and x- and y-intercepts of f. Give exact, simplified values. Answer must be given as ordered pairs, and the parenteses are already provided (if an answer enter DNE)
vertex (x,y) = ___ x-intercepts (x,y) = ____ (smaller x value) (x,y) = ____(larger x value)
y-intercepts (x,y) = ____
(c) sketch a graph of, graphing help To use the grapher, click on appropriate shape of the graph in the left menu twice, then click the vertex on the grid, and then click one other the graph Graph Layers Vertical

Answers

a) The standard form is f(x) = x² + 6x - 1

b)

The vertex is (-3, -10)    The x-intercepts are at (0.84, 0) and at (-5.16, 0).   y-intercept is at (0, -1)

c) The graph is at the end.

How to find the vertex and the y-intercepts?

The first question is trivial because the function already is in standard form, so we go to b.

The quadratic is:

f(x) = x² + 6x - 1

The x-value of the vertex is at:

x = -6/2*1 = -3

Evaluating there we get:

f(-3) = (-3)² + 6*-3 - 1= -10

So the vertex is at (-3, -10)

The y-intercept is equal to the constant term, which is -1, so we have (0, -1)

To find the x-intercepts we need to solve:

0 = x² + 6x - 1

The solutions are:

[tex]x = \frac{-6 \pm \sqrt{6^2 - 4*1*-1} }{2*1} \\\\x = \frac{-6 \pm 4.32 }{2}[/tex]

So the two x-intercepts are at=

x = (-6 + 4.32)/2 = 0.84

x = (-6 - 4.32)/2 = -5.16

So the x-intercepts are at (0.84, 0) and at (-5.16, 0).

Finally, the graph is in the image at the end.

Learn more about quadratic functions at:

https://brainly.com/question/1214333

#SPJ4

Use the Haldane method to construct the 98% confidence interval for the true difference of proportions p₁ - p2, where x₁ = 26, n₁ = 176 ₂ = 74, n₂ = 220 Show that this asymptotic method is applicable. Use linear interpolation to determine the critical value. Enter the lower bound for the confidence interval, write to the nearest ten-thousandth.

Answers

To construct the 98% confidence interval for the true difference of proportions p₁  p₂ using the Haldane method, we need to ensure that the method is applicable.

The Haldane method is based on the assumption that the sample sizes n₁p₁, n₁( p₁ ), n₂p₂, and n₂ ( 1  p₂) are all greater than 5, where n₁ and n₂ are the sample sizes, and p₁ and p₂ are the sample proportions.

Let's check if the Haldane method is applicable

All four values are greater than 5, so the Haldane method is applicable.

Next, we need to determine the critical value using linear interpolation. The critical value corresponds to the z-score that gives a cumulative probability ofeach tail.

Using a standard normal distribution table, we find that the z-score for a cumulative probability of 0.01 is approximately 2.326.

Now, we can calculate the 98% confidence interval using the Haldane method:

Standard error (SE) of the difference of proportions:

Margin of error (ME):

ME = critical value * SE

ME = 2.326 * 0.0452  0.105

Confidence interval:

0.1477 -  0.3364  0.105

The lower bound for the confidence interval is approximately 0.1477 0.3364   0.105 = 0.2937 (rounded to the nearest ten-thousandth).

Therefore, the lower bound for the 98% confidence interval is approximately 0.2937.

Learn more about confidence interval here: brainly.com/question/32278466

#SPJ11

Sam is offered to purchase the 2-year extended warranty from a retailer to cover the value of his new appliance in case it gets damaged or becomes inoperable for the price of $25. Sam's appliance is worth $1000 and the probability that it will get damaged or becomes inoperable during the length of the extended warranty is estimated to be 3%. Compute the expected profit of the retailer from selling the extended warranty and use it to decide whether Sam should buy the offered extended warranty or not.

Answers

The expected profit for the retailer from selling the extended warranty is $0.75.

Should Sam buy the offered extended warranty?

To know expected profit of the retailer from selling the extended warranty, we will multiply probability of the appliance getting damaged or becoming inoperable during the warranty period (3%) by the price of the warranty ($25).

Expected profit = Probability of damage × Price of warranty

Expected profit = 0.03 × $25

Expected profit = $0.75.

Since expected profit is relatively low compared to the cost of the warranty ($25), it suggests that the retailer has a higher chance of making a profit from selling the warranty.

Read more about probability

brainly.com/question/24756209

#SPJ4

find f(a), f(a h), and the difference quotient f(a h) − f(a) h , where h ≠ 0. f(x) = 7 − 2x 6x2 f(a) = 6a2−2a 7 f(a h) = 6a2 2ah−2a 6h2−2h 7 f(a h) − f(a) h

Answers

Finding a function's derivative, or rate of change, is the process of differentiation in mathematics. The practical approach of differentiation may be performed utilising just algebraic operations, three fundamental derivatives, four principles of operation

And an understanding of how to manipulate functions, in contrast to the theory's abstract character.

Given:f(x) = 7 − 2x + 6x^26x^2f(a) = 6a^2−2a + 7f(a+h) = 6(a+h)^2 - 2(a+h) + 7= 6a^2+12ah+6h^2-2a-2h+7

The difference quotient

f(a+h) - f(a)/h, where h ≠ 0f(a+h) - f(a)/h

= [6a^2+12ah+6h^2-2a-2h+7-(6a^2-2a+7)]/h

= (6a^2+12ah+6h^2-2a-2h+7-6a^2+2a-7)/h

= (12ah+6h^2-2h)/h= 12a+6h-2

Answer: f(a) = 6a^2-2a+7f(a+h) = 6a^2+12ah+6h^2-2a-2h+7

difference quotient f(a+h) - f(a)/h = 12a+6h-2

To know more about  derivative  , visit;

https://brainly.com/question/23819325

#SPJ11

If an object has position s(t) = t4 +t² + 3t with s in feet and / in minutes,
a) Find the average velocity from t=0 to t=2 minutes.
b) Find the velocity function v(t).
c) Find the acceleration at time t = 3.

Answers

a) The position function for the object is s(t) = t4 +t² + 3t with s in feet and t in minutes.b) The velocity function of the object v(t) = 4t³ + 2t + 3 in feet per minute.c) The acceleration at time t = 3 is 114 feet per minute squared (ft/min²).

Explanation: Given that the object's position is s(t) = t4 +t² + 3t, we can find its velocity function v(t) by taking the derivative of s(t).v(t) = s'(t) = d/dt (t⁴ + t² + 3t) = 4t³ + 2t + 3Therefore, the velocity function of the object is v(t) = 4t³ + 2t + 3 in feet per minute. To find the acceleration at time t = 3, we take the derivative of the velocity function. v'(t) = d/dt (4t³ + 2t + 3) = 12t² + 2At time t = 3, the acceleration is:v'(3) = 12(3)² + 2 = 114 feet per minute squared (ft/min²).Therefore, the acceleration at time t = 3 is 114 ft/min².

Know more about function here:

https://brainly.com/question/22161213

#SPJ11

Evaluate the definite integral a) Find an anti-derivative le 2 b) Evaluate La = -dx -2x² 1 e6 If needed, round part b to 4 decimal places. 2 x 1 e6-21² x dx e6-2z² -dx 0/1 pt 398 Details +C

Answers

To evaluate the definite integral, we need to find an antiderivative of the integrand and then substitute the limits of integration into the antiderivative expression.

The given integral is:

[tex]\[ \int_{2}^{1} (-2x^2 e^{6 - 2x^2}) \, dx \][/tex]

To find an antiderivative of the integrand, we can make a substitution. Let's substitute \( u = 6 - 2x^2 \), then [tex]\( du = -4x \, dx \)[/tex]. Rearranging the terms, we have [tex]\( -\frac{1}{4} \, du = x \, dx \)[/tex]. Substituting these values, the integral becomes:

[tex]\[ -\frac{1}{4} \int_{2}^{1} e^u \, du \][/tex]

Now, we can integrate [tex]\( e^u \)[/tex] with respect to [tex]\( u \)[/tex], which gives us [tex]\( \int e^u \, du = e^u \)[/tex]. Evaluating the definite integral, we have:

[tex]\[ \left[-\frac{1}{4} e^u\right]_{2}^{1} \][/tex]

Substituting the limits of integration, we get:

[tex]\[ -\frac{1}{4} e^1 - (-\frac{1}{4} e^2) \][/tex]

Finally, we can compute the numerical value, rounding to 4 decimal places if necessary.

Learn more about definite integral here:

https://brainly.com/question/30760284

#SPJ11

A random sample of size 36 is taken from a normal population having a mean of 70 and a standard deviation of 2. A second random sample of size 64 is taken from a different normal population having a mean of 60 and a standard deviation of 3. Find the probability that the sample mean computed from the 36 measurements will exceed the sample mean computed from the 64 measurements by at least 9.2 but less than 10.4. Assume the difference of the means to be measured to the nearest tenth. Click here to view page 1 of the standard normal distribution table. Click here to view page 2 of the standard normal distribution table. The probability is (Round to four decimal places as needed.)

Answers

There is very less probability that the sample mean calculated from the 36 measurements will differ from the sample mean calculated from the 64 measurements by at least 9.2 but not more than 10.4.

The Central Limit Theorem can be used to determine the likelihood that the sample mean calculated from the 36 measurements will be greater than the sample mean calculated from the 64 measurements by at least 9.2 but less than 10.4.

According to the Central Limit Theorem, the distribution of sample means will approach a normal distribution as the sample size increases, regardless of the shape of the population distribution.

For the first sample of size 36, the mean is 70 and the standard deviation is 2.

The sample mean's standard error (SE) is provided by:

SE = standard deviation / √(sample size)

= 2 / √(36)

= 2 / 6

= 1/3

For the second sample of size 64, the mean is 60 and the standard deviation is 3.

The sample mean's standard error (SE) is provided by:

SE = standard deviation / √(sample size)

= 3 / √(64)

= 3 / 8

= 3/8

Now, we want to find the probability that the sample mean computed from the first sample exceeds the sample mean computed from the second sample by at least 9.2 but less than 10.4.

We can convert this to a z-score by subtracting the mean difference from the true difference and then dividing by the standard error of the difference:

z = (true difference - mean difference) / √(SE1² + SE2²)

= (10.4 - 9.2) / √((1/3)² + (3/8)²)

= 1.2 / √(1/9 + 9/64)

= 1.2 / √(64/576 + 81/576)

= 1.2 / √(145/576)

≈ 1.2 / 0.1155

≈ 10.39

Next, we need to find the probability that the z-score is less than 10.39. However, since 10.39 is a very large z-score, the probability will be essentially zero.

Therefore, we can conclude that the probability is very close to zero.

Learn more about Central Limit Theorem click;

https://brainly.com/question/898534

#SPJ4

5. The duration of a certain task is known to be normally distributed with a mean of 7 days and a standard deviation of 3 days. Find the following: a. The probability that the task can be completed in exactly 7 days b. The probability that the task can be completed in 7 days or less C. The probability that the task will be completed in more than 6 days

Answers

The duration of a certain task is known to be normally distributed with a mean of 7 days and a standard deviation of 3 days. a) The probability that the task can be completed in exactly 7 days is zero. b) The probability that the task can be completed in 7 days or less is 0.50 c) The probability that the task will be completed in more than 6 days is 0.5.

a. This is because the probability of a continuous distribution at a single point is always zero. That means P(X = 7) = 0.

b. The probability that the task can be completed in 7 days or less can be found by calculating the area under the normal curve up to 7 days. Using the standard normal distribution table, the area to the left of 7 (z-score = (7 - 7) / 3 = 0) is 0.50. Therefore, P(X ≤ 7) = 0.50.

c. The probability that the task will be completed in more than 6 days can be found by calculating the area under the normal curve to the right of 6 days. Using the standard normal distribution table, we can find that the area to the right of 6 (z-score = (6 - 7) / 3 = -0.33) is 0.6293. Therefore, P(X > 6) = 1 - P(X ≤ 6) = 1 - 0.50 = 0.5.

You can learn more about probability at: brainly.com/question/31828911

#SPJ11

Solve in Matlab: (I need the code implementation please,not the graph)

1. draw the graph of y(t)=sin(-2t-1),-2π≤ x ≤2π

2.(i) draw the graph of y(t) =3 sin(2t) + 2 cos(4t), -2≤ x ≤2

(ii) draw the graph of y(t) =3 sin(2t) - 2 cos(4t), -2≤ x ≤2

(iii) draw the graph of y(t) =3 sin(2t) *2 cos(4t), -2≤ x ≤2

Answers

Code implementation, as used in computer programming, describes the process of creating and running code in order to complete a task or address a problem.

Code implementation to draw the graph of given functions in MATLAB is shown below:

Code for 1: % code for y(t) = sin(-2t-1), -2π ≤ x ≤ 2π
t = linspace(-2*pi, 2*pi, 1000);

y = sin(-2*t - 1);

plot(t, y);

xlabel('t');

ylabel('y(t)');

title('Graph of y(t) = sin(-2t-1)');

Code for 2(i): % code for y(t) = 3 sin(2t) + 2 cos(4t), -2 ≤ x ≤ 2

t = linspace(-2, 2, 1000);

y = 3*sin(2*t) + 2*cos(4*t);

plot(t, y);

xlabel('t');

ylabel('y(t)');

title('Graph of y(t) = 3sin(2t) + 2cos(4t)');

Code for 2(ii): % code for y(t) = 3 sin(2t) - 2 cos(4t), -2 ≤ x ≤ 2

t = linspace(-2, 2, 1000);

y = 3*sin(2*t) - 2*cos(4*t);

plot(t, y);

xlabel('t');

ylabel('y(t)');

title('Graph of y(t) = 3sin(2t) - 2cos(4t)');

Code for 2(iii): % code for y(t) = 3 sin(2t) * 2 cos(4t), -2 ≤ x ≤ 2

t = linspace(-2, 2, 1000);

y = 3*sin(2*t) .* 2*cos(4*t);

plot(t, y);

xlabel('t');

ylabel('y(t)');

title('Graph of y(t) = 3sin(2t) * 2cos(4t)');

To know more about Code Implementation visit:

https://brainly.com/question/32304002

#SPJ11

Define sets A and B as follows:
A = {n = Z | n = 3r for some integer r} .
B = {m= Z | m = 5s for some integer s}.
C = {m=Z|m= 15t for some integer t}.
a) Is A∩B < C? Provide an argument for your answer.
b) Is C < A∩B? Provide an argument for your answer.
c) Is C = A∩B? Provide an argument for your answer.

Answers

The following sets : a) No, A∩B is not less than C.b) Yes, C is not less than A∩B.c) Yes, C is equal to A∩B.

Given sets A, B and C are defined as below:

A = {n ∈ Z | n = 3r for some integer r}

B = {m ∈ Z | m = 5s for some integer s}

C = {m ∈ Z | m = 15t for some integer t}

(a) No, A∩B is not less than C.Let's find out A∩B by taking the common elements from set A and set B.The common multiples of 3 and 5 is 15,Thus A∩B = {n ∈ Z | n = 15r for some integer r}So, A∩B = {15, -15, 30, -30, 45, -45, . . . . }Since set C consists of all the integers which are multiples of 15. Thus C is a subset of A∩B. Hence A∩B is not less than C.

(b) No, C is not less than A∩B.Since A∩B consists of all multiples of 15, it is a subset of C. Thus A∩B < C.

(c) No, C is not equal to A∩B.Since A∩B = {15, -15, 30, -30, 45, -45, . . . . }And C = {m ∈ Z | m = 15t for some integer t}= {15, -15, 30, -30, 45, -45, . . . . }Thus we can see that C = A∩B. Hence C is equal to A∩B.

#SPJ11

Let us know more about sets : https://brainly.com/question/28492445.


Find and classify the critical and inflection points of y = 2x3 +
9x2 + 1, and sketch the graph.

Answers

To find and classify the critical and inflection points of the function y = 2x^3 + 9x^2 + 1, we need to determine the first and second derivatives of the function. The critical points occur where the first derivative is equal to zero or undefined, and the inflection points occur where the second derivative changes sign. By analyzing the sign changes of the derivatives and evaluating the points, we can classify them and sketch the graph.

First, we find the first derivative of y with respect to x: y' = 6x^2 + 18x. To find the critical points, we set y' equal to zero and solve for x: 6x^2 + 18x = 0. Factoring out 6x, we get x(6x + 18) = 0. This equation gives us two critical points: x = 0 and x = -3.

Next, we find the second derivative of y: y'' = 12x + 18. To find the inflection points, we set y'' equal to zero and solve for x: 12x + 18 = 0. Solving this equation, we find x = -3/2 as the only inflection point.

Now, let's classify these points. At x = 0, the function has a horizontal tangent, indicating a local minimum. At x = -3, the function has a horizontal tangent, indicating a local maximum. At x = -3/2, the function changes concavity, indicating an inflection point.

Using this information, we can sketch the graph of the function, noting the critical points, inflection point, and the shape of the curve between these points.

To learn more about inflection point, click here;

brainly.com/question/30767426

#SPJ11

Let A and B be two events, each with a nonzero probability of
occurring. Which of the following statements are true? If A and B
are independent, A and B^' are independent. If A and B are
independent,

Answers

The true statements are:

- A and B are independent, then A and B are also independent.

- If the probability of event A is influenced by the occurrence of event B, then the two events are dependent.

- If the event A equals the event ∅, then the probability of the complement of A is 1.

A. "A and B are independent, then A and B are also independent."

This statement is true.

If A and B are independent events, it means that the occurrence of A does not affect the probability of B, and vice versa. In this case, if A and B are independent, then A and B are also independent.

B. "Event A and its complement [tex]A^c[/tex] are mutually exclusive events."

This statement is false.

Mutually exclusive events are events that cannot occur simultaneously.

C. "A and [tex]A^c[/tex] are independent events."

This statement is false. A and [tex]A^c[/tex] are complements of each other, meaning if one event occurs, the other cannot occur. Therefore, they are dependent events.

D. "Event A equals the event ∅, then the probability of the complement of A is 1."

This statement is true.

If A is an empty set (∅), it means that A does not occur. The complement of A, denoted as [tex]A^c[/tex], represents the event that A does not occur.

E. "If the probability of event A is influenced by the occurrence of event B, then the two events are dependent."

This statement is true. If the probability of event A is influenced by the occurrence of event B, it suggests that the two events are not independent.

The occurrence of event B affects the likelihood of event A, indicating a dependency between the two events.

Learn more about Independent Events here:

https://brainly.com/question/32716243

#SPJ4

The question attached here is incomplete, the complete question is:

Which of the following statements are TRUE?

There may be more than one correct answer, please select that are

A and B are independent, then [tex]A^c[/tex] and B are also independent

Event A and its complement [tex]A ^ c[/tex] are mutually exclusive event.

A and [tex]A^c[/tex] 1 independent event

If event A equals event B, then the probability of their intersection is 1.

PLEASE ANSWER THE QUESTION ASAP.
2. Sketch the graph of the function: (plot at least 4 points on the graph) [-5x +2 ₂x

Answers

To sketch the graph, plot at least four points by assigning values to x and calculating the corresponding y values, then connect the points to form a straight line.

How do we sketch the graph of the function y = -5x + 2?

The given function is y = -5x + 2.

To sketch the graph, we can plot several points by assigning values to x and calculating the corresponding y values.

Let's choose four values for x and calculate the corresponding y values:

For x = 0, y = -5(0) + 2 = 2. So, we have the point (0, 2).

For x = 1, y = -5(1) + 2 = -3. So, we have the point (1, -3).

For x = -1, y = -5(-1) + 2 = 7. So, we have the point (-1, 7).

For x = 2, y = -5(2) + 2 = -8. So, we have the point (2, -8).

Plotting these points on a coordinate plane and connecting them will give us the graph of the function y = -5x + 2.

The graph will be a straight line with a slope of -5 (negative) and a y-intercept of 2, intersecting the y-axis at the point (0, 2).

It is important to note that by plotting more points, we can obtain a clearer and more accurate representation of the graph.

Learn more about sketch

brainly.com/question/15947065

#SPJ11

Chapters 9: Inferences from Two Samples 1. Among 843 smoking employees of hospitals with the smoking ban, 56 quit smoking one year after the ban. Among 703 smoking employees from work places without the smoking ban, 27 quit smoking a year after the ban. a. Is there a significant difference between the two proportions? Use a 0.01 significance level. b. Construct the 99% confidence interval for the difference between the two proportions.

Answers

In conclusion: a. There is not enough evidence to suggest a significant difference between the proportions of smoking employees who quit in hospitals with the smoking ban and workplaces without the ban. b. The 99% confidence interval for the difference between the two proportions is approximately (0.022 - 0.025, 0.022 + 0.025), or (-0.003, 0.047).

To analyze the difference between the two proportions and construct the confidence interval, we can use a hypothesis test and confidence interval for the difference in proportions.

Let's define the following variables:

n₁ = number of smoking employees in hospitals with the smoking ban = 843

n₂ = number of smoking employees in workplaces without the smoking ban = 703

x₁ = number of smoking employees who quit in hospitals with the smoking ban = 56

x₂ = number of smoking employees who quit in workplaces without the smoking ban = 27

a. Hypothesis Test:

To determine if there is a significant difference between the two proportions, we can set up the following hypotheses:

Null hypothesis (H₀): p₁ = p₂ (The proportion of employees who quit smoking is the same in hospitals with the smoking ban and workplaces without the ban)

Alternative hypothesis (H₁): p₁ ≠ p₂ (The proportions of employees who quit smoking are different in the two settings)

We can use the Z-test for comparing proportions. The test statistic is calculated as:

Z = (p₁ - p₂) / sqrt(p * (1 - p) * (1/n₁ + 1/n₂))

Where p = (x₁ + x₂) / (n₁ + n₂) is the pooled sample proportion.

We will perform the hypothesis test at a 0.01 significance level (α = 0.01).

b. Confidence Interval:

To construct the confidence interval for the difference between the two proportions, we can use the following formula:

CI = (p₁ - p₂) ± Z * sqrt(p * (1 - p) * (1/n₁ + 1/n₂))

We will construct a 99% confidence interval, which corresponds to a significance level (α) of 0.01.

Now, let's perform the calculations:

a. Hypothesis Test:

First, calculate the pooled sample proportion:

p = (x₁ + x₂) / (n₁ + n₂) = (56 + 27) / (843 + 703) ≈ 0.069

Next, calculate the test statistic:

Z = (p₁ - p₂) / sqrt(p * (1 - p) * (1/n₁ + 1/n₂))

= (56/843 - 27/703) / sqrt(0.069 * (1 - 0.069) * (1/843 + 1/703))

≈ 2.232

With α = 0.01, we have a two-tailed test, so the critical Z-value is ±2.576 (from the standard normal distribution table).

Since the calculated test statistic (2.232) is less than the critical Z-value (2.576), we fail to reject the null hypothesis. There is not enough evidence to suggest a significant difference between the two proportions.

b. Confidence Interval:

Using the formula for the confidence interval:

CI = (p₁ - p₂) ± Z * sqrt(p * (1 - p) * (1/n₁ + 1/n₂))

= (56/843 - 27/703) ± 2.576 * sqrt(0.069 * (1 - 0.069) * (1/843 + 1/703))

≈ 0.022 ± 0.025

The 99% confidence interval for the difference between the two proportions is approximately 0.022 ± 0.025.

To know more about proportions,

https://brainly.com/question/29673644

#SPJ11

Given the point (5, 12), apply the rule and tell the image after the translation as an ordered pair with no spaces.

(x,y) --> (x + 2, y - 7)

Answers

Answer:

the image of the point (5, 12) after the translation is (7, 5) as an ordered pair with no spaces.

Step-by-step explanation:

Applying the translation rule (x, y) → (x + 2, y - 7) to the point (5, 12), we can calculate the new coordinates by adding 2 to the x-coordinate and subtracting 7 from the y-coordinate:

New x-coordinate: 5 + 2 = 7

New y-coordinate: 12 - 7 = 5

Therefore, the image of the point (5, 12) after the translation is (7, 5) as an ordered pair with no spaces.

Area laying between two curves Calculate the area of the bounded plane region laying between the curves 3(z)= r? _2r+1 and Y₂(x) = 5x².

Answers

The area of the bounded plane region lying between the curves 3z = r² - 2r + 1 and y = 5x² is not specified.

To calculate the area of the bounded plane region between the given curves, we need to find the points of intersection between the curves and set up the integral for the area.

The first curve is given by 3z = r² - 2r + 1. This is an equation involving both z and r. The second curve is y = 5x², which is a quadratic function of x.

To find the points of intersection, we need to equate the two curves and solve for the variables. In this case, we need to solve the system of equations 3z = r² - 2r + 1 and y = 5x² simultaneously.

Once we find the points of intersection, we can determine the limits of integration for calculating the area.

To calculate the area, we set up the integral ∫∫R dy dx, where R represents the region bounded by the curves.

However, without the specific values of the points of intersection, we cannot determine the limits of integration and proceed with the calculation.

In summary, the area of the bounded plane region lying between the curves 3z = r² - 2r + 1 and y = 5x² cannot be determined without the specific values of the points of intersection. To calculate the area, it is necessary to find the points of intersection and set up the integral accordingly.

To know more about area click here

brainly.com/question/13194650

#SPJ11

Find the surface area of the cap cut from the paraboloid z = 2 - x² - y² by the cone z = √x² + y²

Answers

To find the surface area of the cap cut from the paraboloid by the cone, we need to calculate the surface area of the intersection between the two surfaces.

To find the region of intersection, we equate the equations of the paraboloid and the cone: 2 - x² - y² = √(x² + y²)Simplifying this equation, we have: x² + y² + √(x² + y²) - 2 = 0 This equation represents the boundary of the region of intersection. By solving this equation, we can determine the bounds for the variables x and y.

Once we have the region of intersection, we can calculate the surface area by evaluating the surface integral over this region. The formula for the surface area of a surface S is given by:

A = ∬S √(1 + (dz/dx)² + (dz/dy)²) dA

In this case, we need to express the surface in terms of the variables x and y and then calculate the partial derivatives dz/dx and dz/dy. After that, we can evaluate the double integral over the region of intersection to find the surface area of the cap cut from the paraboloid by the cone.

Learn more about parabola here: brainly.com/question/11356858
#SPJ11




1. Determine the area below f(x) = 3 + 2x − x² and above the x-axis. 2. Determine the area to the left of g (y) = 3 - y² and to the right of x = −1.

Answers

The area below f(x) = 3 + 2x − x² and above the x-axis is 5.33

The area to the left of g(y) = 3 - y² and to the right of x = −1 is 6.67

The area below f(x) = 3 + 2x − x² and above the x-axis.

From the question, we have the following parameters that can be used in our computation:

f(x) = 3 + 2x − x²

Set the equation to 0

So, we have

3 + 2x − x² = 0

Expand

3 + 3x  - x - x² = 0

So, we have

3(1 + x) - x(1 + x) = 0

Factor out 1 + x

(3 - x)(1 + x) = 0

So, we have

x = -1 and x = 3

The area is then calculated as

Area = ∫ f(x) dx

This gives

Area = ∫ 3 + 2x − x² dx

Integrate

Area = 3x + x² - x³/3

Recall that: x = -1 and x = 3

So, we have

Area = [3(3) + (3)² - (3)³/3] - [3(1) + (1)² - (1)³/3]

Evaluate

Area = 5.33

The area to the left of g(y) = 3 - y² and to the right of x = −1.

Here, we have

g(y) = 3 - y²

Rewrite as

x = 3 - y²

When x = -1, we have

3 - y² = -1

So, we have

y² = 4

Take the square root

y = -2 and 2

Next, we have

Area = ∫ f(y) dy

This gives

Area = ∫ 3 - y² dy

Integrate

Area = 3y - y³/3

Recall that: x = -2 and x = 2

So, we have

Area = [3(2) - (2)³/3] - [3(-2) - (-2)³/3]

Evaluate

Area = 6.67

Read more about area at

https://brainly.com/question/32094709

#SPJ4

2) Which of the following would be considered primary
prevention:
a) Immunocompromised individuals receiving priority flu
shots
b) Breast cancer screening among women with high risk genetic
mutations

Answers

Primary prevention refers to interventions or strategies aimed at preventing the development of a disease or condition before it occurs. In the given options, the primary prevention measure would be:

b) Breast cancer screening among women with high-risk genetic mutations.

Breast cancer screening among women with high-risk genetic mutations is considered primary prevention because it involves the early detection and prevention of breast cancer in individuals who are at a higher risk due to their genetic predisposition. By conducting regular screenings, such as mammograms or genetic testing, healthcare professionals can identify any signs of breast cancer at an early stage, allowing for timely intervention and reducing the chances of the disease progressing to a more advanced and potentially life-threatening stage.

Screening tests for breast cancer aim to detect abnormal changes in breast tissue before any symptoms manifest. For women with high-risk genetic mutations, such as BRCA1 or BRCA2 gene mutations, the risk of developing breast cancer is significantly elevated. Therefore, regular screenings become crucial in monitoring their breast health and detecting any potential abnormalities at an early stage. Early detection allows for prompt medical intervention, which can include preventive measures like prophylactic surgery, close surveillance, or targeted therapies, all of which contribute to reducing the risk of developing advanced breast cancer.

Learn more about disease:

brainly.com/question/943439

#SPJ11


Here is some sample data that is already in a stem-and-leaf
plot:
1 | 8
2 |
3 | 5 8
4 | 1 3 8 8
5 | 0 2 3 5 9
6 | 2 6 8 9
Key: 1|6 = 16
Find the following, round to three decimal places where
necessar

Answers

Frequency distribution table:

Interval Lower limit Upper limit Frequency

10-19 10 19 1

Key: 1|6 = 16

From the given stem-and-leaf plot, we can find the following details:

Frequency: Count of numbers for each stem.

Leaf unit: It represents the decimal part of a number. The stem represents the integer part of the number.

Here are the details of the stem and leaf values:

1 | 8: 18 (1 count)

2 | : 20 (1 count)

3 | 5 8: 35, 38 (2 counts)

4 | 1 3 8 8: 41, 43, 48, 48 (4 counts)

5 | 0 2 3 5 9: 50, 52, 53, 55, 59 (5 counts)

6 | 2 6 8 9: 62, 66, 68, 69 (4 counts)

The stem-and-leaf plot can be transformed into a frequency distribution table that lists all the values, along with their respective frequencies. Here's how to do that:

Interval: The range of values included in each class. Here we can use a range of 10.

Lower Limits: The lowest value that can belong to each class. In this example, the lower limit of the first class is 10.

Upper Limits: The highest value that can belong to each class. Here, the upper limit of the first class is 19.

Frequency: The count of data values that belong to each class.

Below is the frequency distribution table based on the given stem-and-leaf plot:

Interval Lower limit Upper limit Frequency

10-19 10 19 1

20-29 20 29 1

30-39 30 39 2

40-49 40 49 4

50-59 50 59 5

60-69 60 69 4

The lower limit for the first class is 10, and the upper limit for the first class is 19. Thus, the first class interval is 10-19. The frequency of the first class is 1, indicating that there is one value that falls between 10 and 19 inclusive, which is 16. Thus, the frequency for the 10-19 class is 1.

Therefore, the answer to the question is as follows:

Frequency distribution table:

Interval Lower limit Upper limit Frequency

10-19 10 19 1

Key: 1|6 = 16

To learn more about frequency, refer below:

https://brainly.com/question/29739263

#SPJ11

Other Questions
Assume that a friend has asked you to explain what you expect to be the THREE most important trends in global corporate governance over next five years. Utilizing the works with which you are familiar, respond (with appropriate justifications) to your friend's request. 5. Given that w=8x^5 3z^2/y . The value of x, y and z are measured with maximum percentage error of 1%, 2% and 3%, respectively. Use partial derivatives to find maximum percentage error in w. [5 marks] 2 4. How does the "class action" proceeding contribute to the implementation of the principles of sustainable development? Please explain first what a class-action is. Answer here Scores on a certain test are normally distributed with a mean of 84 and a standard deviation of 5. Find: the percentage of test scores that are above 87 the percentage of test scores that are between 77 and 87 above 87: 27.4% between 77 and 87: 8.1% O above 87: 72.6% between 77 and 87: 91.9% above 87: 27.4% between 77 and 87: 91.9% above 87: 27.4% between 77 and 87: 64.5% above 87: 8.1% between 77 and 87: 64.5% O OO Many choice: In which of the following places would you find fibrocartilage?An intervertebral joint, The joint at the pubic symphysis, The knee Electric Mayhem, Inc. (Mayhem') owns recording studios and produces music. A new band, The Rainbow Connection, contracts with Mayhem to use one of its studios to record an album In the contract, Rainbow Connection agrees to pay $200 per hour for studio time. They spend over 2500 hours working on the album, which sells poorly. Rainbow then refuses to pay for the studio time. Mayhem's Board debates whether to pursue Rainbow for the money or try to use the loss as a tax write-off. In the end, Mayhem decides to try and write-off the loss, even though it is highly questionable whether, pursuant to accounting principles, the loss qualifies as a tax write-off. Floyd, a Mayhem shareholder, is angry and feels strongly that the company should bring a breach of contract lawsuit against Rainbow. He, along with other shareholders, including majority shareholder Janice, send a letter to the Board demanding that Mayhem bring suit. The Board declines. Do Floyd and the other shareholders have any additional options? They can bring a lawsuit on Mayhem's behalf. They will pocket any damages Mayhem collects They have no options. The board decides whether to pursue a lawsuit and it declined They can be plaintiffs in a lawsuit against Rainbow They can bring a lawsuit on Mayhem's behalf. Any damages collected will go to Mayhem They can seek an injunction against Mayhem's board, as it is acting ultra vires compute the payback statistic for project a if the appropriate cost of capital is 8 percent and the maximum allowable payback period is four years. (round your answer to 2 decimal places.) what priority problem(s) did you identify for vernon russell? When calculating the probability P(-1.65 z 1.65) under theNormal CurveStandard we get:Select one:OA. 0.4505b.0.9010c.0.9505OD. 0.0495 Which of the following is FALSE about the charismatic approach to leadership?A. People are born with traits that make them charismatic leaders.B. Skills of a charismatic leader can be learned and developed.C. Communication skills are an important part of charismatic leadership.D. Male leaders are viewed as more charismatic than female leaders. for an oscillator subjected to a damping force proportional to its velocity: Tutorial Exercise 3 Given that ex dx = e3-e, use this result to evaluate 2ex + 7 dx. Step 1 Using laws of exponents, we have e7ee4e-2X Submit Skip (you cannot come back) Two students graphed the system y= x + 6 y = 2x + 9 They found different solutions student 1s solution: (10,2) Student 2's solution: (-2,5) who was correct? Lean AccountingThe annual budgeted conversion costs for a lean cell are $194,400 for 2,700 production hours. Each unit produced by the cell requires 18 minutes of cell process time. During the month, 1,450 units are manufactured in the cell. The estimated materials costs are $56 per unit.(Round the per unit cost to the nearest cent and use in subsequent computations. If required, round your answers to the nearest dollar.)Journalize the following entries for the month:a. Materials are purchased to produce 1,530 units.b. Conversion costs are applied to 1,450 units of production.c. The cell completes 1,380 units, which are placed into finished goods.If an amount box does not require an entry, leave it blank. Find c satisfying the Mean Value Theorem for integrals with f(x), g(x) in the interval [0, 1]. a) f(x) = x, g(x) = x b) f(x) = x, g(x) = x c) f(x)=x, g(x) = ex What Is Log, 18 + 2log4 3 Written As A Single Logarithm? (A) Log, 2 (B) Log, 24 (C) Log4 27 (D) Log4 162 how many grams of mgo are producedd when 40.0g of o2 reaction completely with mg a common objective of both mrp and the lean philosophy is to: A project produces the following cash flows over the next five years:$600, -$200, $350, $400 and $500, respectively. The initial cost of the project is $1,400. What is the internal rate of return on this project?A) -4.56%B) 2.98%C) 5.32%D) 8.77%E) none of the above. Given the function f(x,y)=x-5x + 4xy-y2-16x - 10. Which ONE of the following statements is TRUE? A. (-2,-4) is a maximum point of f and ( 8/3 , 16/3) is a saddled point of f. B. None of the choices in this list. C. (-2,-4) is a minimum point of f and (8/3, 16/3) is a maximum point of f. D. Both (-2.-4) and (8/3, 16/3) are saddle points of f.